Calculating Constant C in a Differential Equation with Initial Condition

Click For Summary
The discussion centers on solving a differential equation with an initial condition of x(π) = 2. The general solution derived is x(t) = (1/9t) sin(3t) - (1/3) cos(3t) + c/t. The main query involves whether to use radians or degrees for the calculations, with consensus that radians should always be used in calculus. Participants confirm the correctness of the general solution and emphasize that using degrees complicates the differentiation of trigonometric functions. The thread concludes with a clear understanding of the need for radians in this context.
roam
Messages
1,265
Reaction score
12

Homework Statement



http://img546.imageshack.us/img546/1860/dequestion.jpg

The Attempt at a Solution



My main problem is with the second part where it says find a solution to the DE that satisfies the initial condition x(π) = 2. I found the general solution to be

x(t) = \frac{1}{9 t} \ sin \ 3t - \frac{1}{3} \ cos \ 3t + \frac{c}{t}

So when we substitute in we get

x(\pi) = 2 = \frac{1}{9 \pi} \ sin \ 3 \pi - \frac{1}{3} \ cos \ 3 \pi + \frac{c}{\pi}

Now I need to solve for the constant C. So my question is: Do I need to calculate this using radians or degrees? I mean, should I have my calculators settings on degrees or radians? :confused:

By the way this is how I solved the DE:

\frac{dx}{dt} + \frac{x}{t} = sin \ 3t

using the integrating factor

\mu (t) = e^{\int \frac{1}{t} dt}= k \ e^{\ln |t|} = t

t \frac{dx}{dt} + t \frac{x}{t}= t(sin \ 3t)

\int \frac{d}{dt} tx = \int t \ sin \ 3t

Using integration by parts for the RHS

tx = \frac{1}{9} \ sin \ 3t - \frac{t}{3} \ cos \ 3t + k

\therefore \ x(t) = \frac{1}{9 t} \ sin \ 3t - \frac{1}{3} \ cos \ 3t + \frac{c}{t}

Is this correct? I think I solved it correctly, but I would still appreciate it if anyone could let me know if there are any mistakes. I'm a bit unsure because most of the DE solutions I've seen don't have a variable in the denominator like the 1/9t term.
 
Last edited by a moderator:
Physics news on Phys.org
hi roam! :wink:

yes, your general solution is fine :smile:
roam said:
Now I need to solve for the constant C. So my question is: Do I need to calculate this using radians or degrees? I mean, should I have my calculators settings on degrees or radians? :confused:

you always use radians

always always always! :biggrin:

(though you shouldn't need a calculator for cos3π and sin3π :wink:)
 
roam said:

Homework Statement



http://img546.imageshack.us/img546/1860/dequestion.jpg

The Attempt at a Solution



My main problem is with the second part where it says find a solution to the DE that satisfies the initial condition x(π) = 2. I found the general solution to be

x(t) = \frac{1}{9 t} \ sin \ 3t - \frac{1}{3} \ cos \ 3t + \frac{c}{t}

So when we substitute in we get

x(\pi) = 2 = \frac{1}{9 \pi} \ sin \ 3 \pi - \frac{1}{3} \ cos \ 3 \pi + \frac{c}{\pi}

Now I need to solve for the constant C. So my question is: Do I need to calculate this using radians or degrees? I mean, should I have my calculators settings on degrees or radians? :confused:

By the way this is how I solved the DE:

\frac{dx}{dt} + \frac{x}{t} = sin \ 3t

using the integrating factor

\mu (t) = e^{\int \frac{1}{t} dt}= k \ e^{\ln |t|} = t

t \frac{dx}{dt} + t \frac{x}{t}= t(sin \ 3t)

\int \frac{d}{dt} tx = \int t \ sin \ 3t

Using integration by parts for the RHS

tx = \frac{1}{9} \ sin \ 3t - \frac{t}{3} \ cos \ 3t + k

\therefore \ x(t) = \frac{1}{9 t} \ sin \ 3t - \frac{1}{3} \ cos \ 3t + \frac{c}{t}

Is this correct? I think I solved it correctly, but I would still appreciate it if anyone could let me know if there are any mistakes. I'm a bit unsure because most of the DE solutions I've seen don't have a variable in the denominator like the 1/9t term.

Always use radians in calculus. The formulas become quite messy if you use degrees. For example, if we measure the angle x in degrees we have (d/dx) sin(x) = (π/180)*cos(x) and (d/dx) cos(x) = -(π/180)*sin(x).

RGV
 
Last edited by a moderator:
Ah I see! Thanks a lot tiny tim and Ray for your responses, I get it now!
 
Question: A clock's minute hand has length 4 and its hour hand has length 3. What is the distance between the tips at the moment when it is increasing most rapidly?(Putnam Exam Question) Answer: Making assumption that both the hands moves at constant angular velocities, the answer is ## \sqrt{7} .## But don't you think this assumption is somewhat doubtful and wrong?

Similar threads

  • · Replies 11 ·
Replies
11
Views
3K
  • · Replies 1 ·
Replies
1
Views
2K
  • · Replies 3 ·
Replies
3
Views
2K
Replies
8
Views
3K
Replies
1
Views
1K
  • · Replies 2 ·
Replies
2
Views
2K
  • · Replies 105 ·
4
Replies
105
Views
6K
  • · Replies 3 ·
Replies
3
Views
2K
Replies
1
Views
2K
  • · Replies 5 ·
Replies
5
Views
2K